0 Daumen
1,2k Aufrufe

Aufgabe:

Grenzwert der Folge bestimmen.

\(\sum\limits_{n=1}^{\infty} \frac{1}{n}sin(n\frac{\pi}{2}) \)


Problem/Ansatz:

Laut wolfram ist das Ergebnis π/4.

und sin(n pi/2) ist ja -1, 0 oder 1. wie kann ich das als (-1)^... ausdrücken, falls nötig?

Avatar von

1 Antwort

+2 Daumen
 
Beste Antwort

Hallo,

\(\sin\left(\frac{n\pi}{2}\right)=1\) für \(n=4k+1\)

\(\sin\left(\frac{n\pi}{2}\right)=-1\) für \(n=4k+3\)

\(\sin\left(\frac{n\pi}{2}\right)=0\) sonst.

Damit reduziert sich die Reihe zu \(\sum_{n=1}^{\infty} \frac{(-1)^{n+1}}{2n-1}\).

Schreibt man \(\frac{1}{1-2n}=\int_{0}^{1}x^{-2n}\, dx\), so folgt:$$\sum_{n=1}^{\infty} \frac{(-1)^{n+1}}{2n-1}=\sum_{n=1}^{\infty} (-1)^{n+1}\cdot (-1)\cdot \int_{0}^{1}x^{-2n}\, dx$$ Vertausche nun Integrationszeichen mit der Reihe und wende die geometrische Reihenformel an.

Avatar von 28 k

danke und ich habe noch paar fragen:

wie kommt man von 4k+1 auf n+1? ist n = 4k? und wir benutzen hier die obere grenze von 1 und nicht -1? und 0 wird ignoriert, da wir n=4k gesagt haben?

wieso schreibt man  \( \frac{1}{1-2n}=\int_{0}^{1}x^{-2n}\, dx\)

sollte das nicht \( \int_{0}^{1}-x^{-2n}\, dx = \frac{1}{2n-1}\) sein?

mfg

Hallo,

um auf \(\sum_{n=1}^{\infty} \frac{(-1)^{n+1}}{2n-1}\) zu kommen, schreibt man einfach mal ein paar Glieder auf:$$\sum\limits_{n=1}^{\infty} \frac{1}{n}\sin(n\frac{\pi}{2})=1-\frac{1}{3}+\frac{1}{5}-\frac{1}{7}\pm \cdots$$ Der Nenner wächst wie \(2n-1\) und der Zähler alterniert.

Nein, \(\frac{1}{1-2n}=\int_{0}^{1}x^{-2n}\, dx\) hat schon seine Richtigkeit. Vgl. WolframAlpha.

Sicher, dass 1/4 in den Gliedern passt?

Nein, natürlich nicht. Habe es editiert.

Danke, das hat mein feng shui ganz durcheinander gebracht ;)

Ich gebe dir mal ein +1, weil ich deine Antwort gut finde, auch wenn ich die Frage nicht gestellt habe. Was mich etwas stört ist dieses "herumfrickeln" mit den alternierenden Gliedern, entweder sieht man den mathematischen Zusammenhang oder nicht, hier war das ja noch möglich, aber es gibt weitaus schlimmeres :)

Geheimtipp: Bei WolframAlpha kann man schummeln (hihi). WolframAlpha findet dann das wahrscheinlichste Bildungsgesetz

blob.png

In diesem Beispiel war das Bildungsgesetz aber wirklich noch gut zu erkennen.

wieso gilt das? dein integral ist richtig. das mein ich aber nicht.

\( \sum_{n=1}^{\infty} \frac{(-1)^{n+1}}{2n-1}=\sum_{n=1}^{\infty} (-1)^{n+1}\int_{0}^{1}x^{-2n}\, dx\)

die rechte seite ist doch eindeutig \( \sum_{n=1}^{\infty} \frac{(-1)^{n+1}}{1-2n}\) und das ist was anderes

Hallo,

gut erkannt. Da ist mir in der Tat ein Minus verloren gegangen. Es gilt \(\frac{1}{2n-1}=-\frac{1}{1-2n}\) also \(\frac{1}{2n-1}=-\int_{0}^{1}x^{-2n}\, dx\)

-1^n wird mit dem rest zu -q^n der geometr. reihe?

was passiert mit dem - vor dem integral?

und was mich noch irritiert ist was mit dem ^-2 bei x passiert. also ^-2 wird dann zu ^2 bei der geometr. reihe.


mfg

Wir machen es so: Ich schreibe den Rechenweg noch einmal klarer (auch mit Erklärungen) auf und dann melde ich mich nochmal.

es ist schon alles ist ordnung. nur noch ein problem.

bei -x^(-2). das wird in der geometrischen reihe zu 1/1-(-x^(+2))?

wieso wird ^-2 zu ^+2?


mfg

Oh, entschuldige, ich war gestern noch außer Haus und habe es dann vergessen.

Es gilt doch \(-x^{-2}=-\frac{1}{x^2}\). Das ist einfach ein Potenzgesetz (x^(-1)=1/x).

Wie kommst du auf diese Umrechnung: $$(1)\sum \limits_{n=0}^{\infty}\frac{(-1)^{n+1}}{2n-1} = \sum \limits_{n=0}^{\infty}(-1)^{n+1}\int_{0}^{1}x^{-2n}dx$$


$$\int_{0}^{1}\sum \limits_{n=0}^{\infty}(-x^2)^n dx = \int_{0}^{1}\frac{1}{x^{2}+1}=\frac{π}{4}$$


Könntest du bitte die Schritte näher erklären?


Und wie kommt man auf diese Lösung (1)?

Und wie bist du auf diese Reihe genau gekommen(ohne Hilfsmittel)?

Die Reihe kann man doch wirklich raten!:$$\sum\limits_{n=1}^{\infty} \frac{1}{n}\sin(n\frac{\pi}{2})=1-\frac{1}{3}+\frac{1}{5}-\frac{1}{7}\pm \cdots$$ Man sieht doch eindeutig, dass der Nenner wie \(2n-1\) wächst und der Zähler immer alterniert!

Nenner: 1,3,5,7,...

Zähler: 1,-1,1,-1,...

Kannst du die Zahlenreihen erkennen?

$$(1)\sum \limits_{n=0}^{\infty}\frac{(-1)^{n+1}}{2n-1} = \sum \limits_{n=0}^{\infty}(-1)^{n+1}\int_{0}^{1}x^{-2n}dx$$Das ist Produkt von Kreativität, Erfahrung und Herumprobieren.

Die Vertauschung von \(\int\) und \(\Sigma\) ist ein starkes Werkzeug, um Reihen mit denen man nichts anfangen kann, zu bändigen. Versuch einfach immer einen Teil als Integral auszudrücken.

Und wie hast du den Nenner herausgefunden bzw. vereinfacht (-1)^(n+1)

(-1)^(n+1) wird für gerade n negativ und für ungerade n positiv.$$\sum\limits_{n=1}^{\infty} \frac{1}{n}\sin(n\frac{\pi}{2})=1-\frac{1}{3}+\frac{1}{5}-\frac{1}{7}\pm \cdots$$ Wir haben also für das erste Reihenglied (1 ist ungerade) ein positives Vorzeichen. Für das zweite Reihenglied ein negatives Vorzeichen. Für das dritte Reihenglied ein positives. Für das vierte ein negatives

....

...

..

(x^(-1)=1/x)

das ist ne normale potenz. das ist mir schon klar.

ich habe vergessen, dass das eine summe 1 bis unendlich ist.

da muss ich ja noch den 0 ten glied abziehen. das war wohl der fehler. nun kommt bei mir auch dasselbe raus.


vielen dank!

zu seiner Frage. da fehlt noch eine -.

Vielleicht sollte man das noch editieren, damit keine weiteren probleme aufkommen, wenn andere leute wieder hier landen.

das war bei mir ja auch ein Problem, weil das - fehlte.

also -x^(-2n) muss dahin.

ich komme nicht auf die form \(\frac{1}{x^2+1}\) das integriert von 0 bis 1 ergibt \(\frac{\pi}{4}\).

Das ist meine Rechnung:

\( =-(-1)^{1} \int_{0}^{1} \sum \limits_{n=1}^{\infty}(-1)^{n}\left(x^{-2}\right)^{n} d x \)

\( =-\sum \limits_{n=1}^{\infty}\left(x^{-2}\right)^{n} \)

\( =-\frac{1}{1-q} \)
\( =-\left(\frac{1}{1-x^{-2}}-x^{2 \cdot 0}\right) \)
\( =-\left(\frac{x^{2}}{x^{2}-1}-\frac{x^{2}-1}{x^{2}-1}\right) \)
\( =-\left(\frac{1}{x^{2}-1}\right) \)
\( = \frac{1}{1-x^{2}} \)
\( = \int_{0}^{1} \frac{1}{1-x^{2}} \neq \frac{\pi}{4}\)

wo ist der Fehler?

ok das hat sich geklärt

Für diejenigen, die es interessiert, es ist:

$$\int \frac{1}{1+x^2} \, dx = \tan^{-1}(x)$$

Nun ist $$\tan^{-1}(0) = 0$$ und $$\tan^{-1}(1) = \frac{\pi}{4}$$

Integralgrenzen sind 0, 1...

FIN!

Und wie bist du auf 4k gekommen?

4k/2 =2k

2k * pi ist die Periode des Sinus

Alles das findest du unter

" Leibniz-Reihe"

Hallo,

kann mir mal einer das uneigentliche Integral

$$\int_0^1 \frac{1}{x^{2n}} dx$$

vorrechnen,

Gruß

Ja, schreib das mal via Potenzgesetz um in $$\frac{1}{x^{2n}} = x^{-2n}$$, dann einfach "wie gewohnt" integrieren über die Potenzregel: $$\int x^{n}\,dx=\frac{x^{n+1}}{n+1}$$

Hallo,

ich würde gerne sehen, wie Du \(x=0\)  in \(x^{-2n+1}\), alos zum Beispiel in \(x^{-1}\)  einsetzt.

Gruß

Ein anderes Problem?

Stell deine Frage

Willkommen bei der Mathelounge! Stell deine Frage einfach und kostenlos

x
Made by a lovely community